Is the Inequality $(a+b+c)^2+(x+y+z)^2≥1$ Always True?

  • MHB
  • Thread starter anemone
  • Start date
In summary, the given conditions imply that the points $P,Q,R$ all lie on the upper half of the unit circle. Using a geometric approach, it can be proven that $(a+b+c)^2+(x+y+z)^2 > 1$, with equality only occurring in the limiting case. Therefore, the inequality $(a+b+c)^2+(x+y+z)^2 \geq 1$ holds true.
  • #1
anemone
Gold Member
MHB
POTW Director
3,883
115
Let $x,\,y,\,z>0$ and $a,\,b,\,c$ be real numbers such that $x^2+a^2=y^2+b^2=z^2+c^2=1$.

Prove that $(a+b+c)^2+(x+y+z)^2\ge 1$.
 
Mathematics news on Phys.org
  • #2
anemone said:
Let $x,\,y,\,z>0$ and $a,\,b,\,c$ be real numbers such that $x^2+a^2=y^2+b^2=z^2+c^2=1$.

Prove that $(a+b+c)^2+(x+y+z)^2\ge 1$.
[sp]Here is a geometric approach to the problem.
[TIKZ]\clip (-6,-1) rectangle (12,12) ;
\draw [thin] (-6,0) -- (6,0) ;
\draw [thin] (0,-1) -- (0,10) ;
\coordinate [label=above left:{$P$}] (P) at (-4,3) ;
\coordinate [label=above right:{$Q$}] (Q) at (1.92,4.62) ;
\coordinate [label=right:{$R$}] (R) at (4.42,2.35) ;
\draw [thin,gray] (P) -- (3.17,3.5) ;
\draw [thin,gray] (Q) -- (0.21,2.675) ;
\draw [thin,gray] (R) -- (-1.04,3.81) ;
\coordinate [label=below:{$C$}] (C) at (0,0) ;
\coordinate [label=right:{$G$}] (G) at (0.78,3.33) ;
\coordinate [label=right:{$H$}] (H) at (2.34,10) ;
\coordinate [label=below:{$N$}] (N) at (1.75,2.55) ;
\draw [thin,gray] (H) -- (N) ;
\draw [red] (C) --(G) -- (H) ;
\foreach \point in {C,G,H}
\fill (\point) circle(2pt) ;
\draw (0,0) circle (5cm) ;
\draw (P) -- (Q) -- (R) -- cycle ;
[/TIKZ]​
Let $P = (a,x)$, $Q = (b,y)$ and $R = (c,z)$. The given conditions imply that $P,Q,R$ all lie on the upper half of the unit circle. Relabelling the points if necessary, we may assume that $Q$ lies on the arc between $P$ and $R$. Notice that the angle $PQR$ is obtuse.

The origin $C$ is the circumcentre of the triangle $PQR$. Let $G = \bigl(\frac13(a+b+c),\frac13(x+y+z)\bigr)$ be the centroid of the triangle, and let $H$ be the orthocentre (where the three perpendiculars from the vertices to the opposite sides meet). In particular, let $N$ be the point where the perpendicular from $Q$ meets the side $PR$. The fact that the angle at $Q$ is obtuse means that $N$ lies inside the unit circle on the segment $PR$, and $H$ lies outside the circle, on the opposite side of $Q$ from $N$. Therefore the distance $|CH|$ is greater than $1$.

The theory of the Euler line tells us that the points $C,G,H$ are collinear and that $|GH| = 2|CG|$, so that $|CH| = 3|CG|$. Since $G = \bigl(\frac13(a+b+c),\frac13(x+y+z)\bigr)$, it follows that $H = \bigl((a+b+c),(x+y+z)\bigr)$. But then the fact that $|CH| >1$ tells us that $(a+b+c)^2 + (x+y+z)^2 > 1.$[/sp]
 
  • #3
Opalg said:
[sp]Here is a geometric approach to the problem.
[TIKZ]\clip (-6,-1) rectangle (12,12) ;
\draw [thin] (-6,0) -- (6,0) ;
\draw [thin] (0,-1) -- (0,10) ;
\coordinate [label=above left:{$P$}] (P) at (-4,3) ;
\coordinate [label=above right:{$Q$}] (Q) at (1.92,4.62) ;
\coordinate [label=right:{$R$}] (R) at (4.42,2.35) ;
\draw [thin,gray] (P) -- (3.17,3.5) ;
\draw [thin,gray] (Q) -- (0.21,2.675) ;
\draw [thin,gray] (R) -- (-1.04,3.81) ;
\coordinate [label=below:{$C$}] (C) at (0,0) ;
\coordinate [label=right:{$G$}] (G) at (0.78,3.33) ;
\coordinate [label=right:{$H$}] (H) at (2.34,10) ;
\coordinate [label=below:{$N$}] (N) at (1.75,2.55) ;
\draw [thin,gray] (H) -- (N) ;
\draw [red] (C) --(G) -- (H) ;
\foreach \point in {C,G,H}
\fill (\point) circle(2pt) ;
\draw (0,0) circle (5cm) ;
\draw (P) -- (Q) -- (R) -- cycle ;
[/TIKZ]​
Let $P = (a,x)$, $Q = (b,y)$ and $R = (c,z)$. The given conditions imply that $P,Q,R$ all lie on the upper half of the unit circle. Relabelling the points if necessary, we may assume that $Q$ lies on the arc between $P$ and $R$. Notice that the angle $PQR$ is obtuse.

The origin $C$ is the circumcentre of the triangle $PQR$. Let $G = \bigl(\frac13(a+b+c),\frac13(x+y+z)\bigr)$ be the centroid of the triangle, and let $H$ be the orthocentre (where the three perpendiculars from the vertices to the opposite sides meet). In particular, let $N$ be the point where the perpendicular from $Q$ meets the side $PR$. The fact that the angle at $Q$ is obtuse means that $N$ lies inside the unit circle on the segment $PR$, and $H$ lies outside the circle, on the opposite side of $Q$ from $N$. Therefore the distance $|CH|$ is greater than $1$.

The theory of the Euler line tells us that the points $C,G,H$ are collinear and that $|GH| = 2|CG|$, so that $|CH| = 3|CG|$. Since $G = \bigl(\frac13(a+b+c),\frac13(x+y+z)\bigr)$, it follows that $H = \bigl((a+b+c),(x+y+z)\bigr)$. But then the fact that $|CH| >1$ tells us that $(a+b+c)^2 + (x+y+z)^2 > 1.$[/sp]
but how can it be: $(a+b+c)^2+(x+y+z)^2=1\,\, ?$
for the question :$(a+b+c)^2+(x+y+z)^2\geq1 $
when will the equality happen ?
 
Last edited:
  • #4
Since $x^2+a^2=1,$ therefore, $0<x\le1 $ and $|a|<1$ by the given conditions($x,y,z,a,b,c\in\mathbb{R}$). Similarly, $0<y,z\le1$ and $|b|,|c|<1$. Now, $$(x+y+z)^2+(a+b+c)^2=3+2(xy+yz+zx)+2(ab+bc+ca)\ge3+2(ab+bc+ca)$$ because of the positivity of $x,y,z$. Now, atleast one of $ab,bc$ or $ca$ must be positive and the minimum value of the sum of other two approaches $-2$. Note that when the negative products approach $-2$, the positive product approaches $1$.Therefore, $(x+y+z)^2+(a+b+c)^2\gt3+2(1)-2(2)=1$.
 
Last edited by a moderator:
  • #5
Albert said:
but how can it be: $(a+b+c)^2+(x+y+z)^2=1\,\, ?$
for the question :$(a+b+c)^2+(x+y+z)^2\geq1 $
when will the equality happen ?
[sp]Equality can only happen in the limiting case where (in the diagram in my previous comment) $P$ and $Q$ are on the $x$-axis at the points $\pm1$. But the question stipulates that $x$, $y$ and $z$ are strictly positive. That implies that the limiting case cannot occur, and so the conclusion can be strengthened to $(a+b+c)^2+(x+y+z)^2 > 1 $.[/sp]
 

Related to Is the Inequality $(a+b+c)^2+(x+y+z)^2≥1$ Always True?

1. What is the meaning of the inequality (x+y+z)²+(a+b+c)²≥1?

The inequality (x+y+z)²+(a+b+c)²≥1 means that the sum of the squares of three variables x, y, and z plus the sum of the squares of three other variables a, b, and c is greater than or equal to 1.

2. How can I prove that (x+y+z)²+(a+b+c)²≥1 is true?

To prove this inequality, you can use the Cauchy-Schwarz inequality which states that for any two sets of numbers, the sum of the squares of their products is always greater than or equal to the square of the sum of the products. In this case, you can apply this inequality to the sets (x, y, z) and (1, 1, 1), and also to the sets (a, b, c) and (1, 1, 1) to show that the given inequality is true.

3. Can you provide an example to illustrate the inequality (x+y+z)²+(a+b+c)²≥1?

Sure, for example, let x = 2, y = 3, z = 4, a = 0, b = 1, and c = -2. Then, (x+y+z)²+(a+b+c)² = (2+3+4)² + (0+1+(-2))² = 81 + 1 = 82, which is greater than or equal to 1.

4. How does the inequality (x+y+z)²+(a+b+c)²≥1 relate to the Pythagorean theorem?

The inequality (x+y+z)²+(a+b+c)²≥1 is similar to the Pythagorean theorem, which states that in a right triangle, the square of the length of the hypotenuse is equal to the sum of the squares of the lengths of the other two sides. In this case, the inequality shows that the sum of the squares of two sets of numbers is greater than or equal to the square of the sum of those numbers, which is similar to the Pythagorean theorem.

5. How is the inequality (x+y+z)²+(a+b+c)²≥1 useful in mathematics?

The inequality (x+y+z)²+(a+b+c)²≥1 is useful in mathematics because it is a fundamental concept that is used in various fields, such as algebra, geometry, and statistics. It is also used to prove other important theorems and inequalities, and it helps to establish relationships between different mathematical concepts.

Similar threads

Replies
1
Views
912
  • General Math
Replies
1
Views
1K
Replies
9
Views
1K
Replies
1
Views
1K
Replies
4
Views
1K
  • General Math
Replies
3
Views
1K
Replies
1
Views
997
Replies
1
Views
910
  • General Math
Replies
1
Views
947
Back
Top